10
$\begingroup$

Suppose we have an arbitrary probability space $(\Omega,\mathcal{F},\mathbb{P})$ and a sequence of real random variables $X_n:\Omega\to\mathbb{R}$ such that the pushforward measures $(X_n)_*(\mathbb{P})$ converge weakly to a probability $\tilde{\mathbb{P}}$.
Can we always construct a random variable $X:\Omega\to\mathbb{R}$ such that $\tilde{\mathbb{P}}=X_*(\mathbb{P})$? (i.e. such that $X_n\to X$ in distribution?)

(I know this is not the kind of questions that should be asked here, but I received no helpful answers on StackExchange..)

$\endgroup$
1
  • $\begingroup$ is there any reference book or paper about this problem? $\endgroup$
    – Dongwei
    Feb 4 at 4:11

1 Answer 1

7
$\begingroup$

Yes I think so. We may assume that $\mathcal F$ is the $\sigma$-algebra generated by sets of the form $X_n^{-1}(a,\infty)$ as $n$ runs over the positive integers and $a$ runs over the reals as $\mathcal F$ already contains these sets. In so doing, we might be making $\mathcal F$ smaller, but that only makes the problem harder.

Let $\mathcal P_1,\mathcal P_2,\mathcal P_3,\ldots$ be a sequence of refining finite partitions of $\mathbb R$ such that the intersection of an element of $\mathcal P_n$ for each $n$ consists of at most one point. For example, $\mathcal P_n$ could consist of intervals $[n,\infty)$, $(-\infty,-n)$ and half-open dyadic intervals of length $1/2^n$ between $-n$ and $n$.

Then let $\mathcal F_n$ be the finite sub-algebra of $\mathcal F$ generated by the sets $X_i^{-1}A$ for $1\le i\le n$ and $A$ running over $\mathcal P_n$. Write $\mathcal F_n=\{B^n_1,\ldots B^n_{k_n}\}$. Now there is a measurable map $\Phi$ from $\Omega$ to $\Xi=\prod_{n=1}^\infty\{1,\ldots,k_n\}$ sending $\omega$ to sequence of partition elements that it lies in.

Equip $\Xi$ with the lexicographic ordering. We can then check that $\mathbb P$ induces a measure $\mu$ on $\Xi$. There are at most countably many atoms in $\Omega$ with respect to $\mathcal F$. Let the set of atoms be $A$. We can map each of these atoms disjointly to atoms of the same mass (at least) in the limit measure $\tilde {\mathbb P}$ (some justification needed here, but I'm pretty confident).

First we couple the atoms in both $\tilde{\mathbb P}$ and $\Omega$. That is we define $X$ restricted to the atoms. To finish, use quantile coupling to couple what's left. Given an $\omega$, set $t(\omega)=\mu(\{\omega'\in \Omega\setminus A\colon \Phi(\omega')\le \Phi(\omega)\})$ and finally, define for $\omega$ not in an atom, $X(\omega)=\inf\{s\colon \big(\tilde{\mathbb P}(-\infty,s])-\sum\text{(atoms of $\tilde{\mathbb P}\le s$)} \big)\ge t\}$.

$\endgroup$
4
  • 3
    $\begingroup$ Basically what you said can be recast as follows: first, there are countably many atoms in $\mathcal F,P$ and we can pass to a subsequence of $X_k$ converging almost surely on each atom, which will imply that the push-forwards also converge weakly to some part of the limiting measure. After that, we are left with non-atomic measure and $\sigma$-field, which can carry a function with any distribution of equal total mass, so you choose one that fits ignoring $X_k$ completely. Neat! Note that I made the same mistake as Ofer and thought that we can change the probability but not the set. $\endgroup$
    – fedja
    Oct 19, 2013 at 12:55
  • $\begingroup$ I like the idea of passing to a subsequence taking care of the atoms. I think this simplifies things a bit. $\endgroup$ Oct 19, 2013 at 18:43
  • $\begingroup$ Very nice! Thanks to fedja too, for his idea. When you talk about atoms, you are identifying atoms such that their symmetric difference is a null set, right? $\endgroup$
    – Mizar
    Oct 19, 2013 at 22:36
  • $\begingroup$ Yeah, it's indeed always possible to construct such an RV. I posted a thorough solution over at math.stackexchange.com/questions/519999/…. Ultimately, very similar ideas to what you wrote about. And yes, I passed to a subsequence to ensure convergence on atoms.Great question. I learned a lot proving it. $\endgroup$ Oct 20, 2013 at 19:06

Your Answer

By clicking “Post Your Answer”, you agree to our terms of service and acknowledge you have read our privacy policy.

Not the answer you're looking for? Browse other questions tagged or ask your own question.